Тёмный

Need Your help to solve this Physics problem!  

ISAC Learning (NMS Sir)
Подписаться 58 тыс.
Просмотров 3,1 тыс.
50% 1

In this video, NMS sir talks about a physics question asked by one of the JEE students at the offline centre.
He explained the question in detail, going step by step so that students could understand how to solve it and Sir also said that if anyone solves it on his own pls suggest their methods in the comment box.
This is helpful for students preparing for the JEE exam, as it teaches problem-solving methods and key concepts that are important for the mock test and JEE Mains exam.
In this question, the electric field and charges are arranged in the form of an equilateral triangle. You are asked to find the angle of deviation.
This is a confusing question, but sir has solved it. However, sir wants everyone to try solving it in their own way. Please attempt it and share your approach to the solution.
Share this JEE Physics problem with everyone and encourage them to solve it. This interesting JEE Physics problem on electric fields and charges will definitely help clear your concepts.
#iitjee #jeemains #jee2025 #jee2026 #physics #nmssir

Опубликовано:

 

26 сен 2024

Поделиться:

Ссылка:

Скачать:

Готовим ссылку...

Добавить в:

Мой плейлист
Посмотреть позже
Комментарии : 24   
@RajVanshSingh1904
@RajVanshSingh1904 6 дней назад
One key thing to understand here is that in three dimensions, there isn’t any symmetry to rely on. In the case of two charges, we frequently use solid angles when considering them because their symmetry tells that the behavior is uniform in every direction from the line joining them. However when we return to considering three charges, for each charge a different plane is defined. Everything in that particular plane is not what happens in any other plane. This could be a very nice explanation but in “your frame” where you see field lines coming out from charges might not seem the same going out of charge in another orientation. But if you say that each charge emits N lines, notice that this amount will not be just Q/ϵ because the lines spread out to every direction, including perpendicular to the screen. Therefore, N is just a dimensionless proportional factor in this two dimensional problem. This has been addressed by PHYSICS SIR JEE - JANARDHAN in one of his videos for " ANGLE OF DEVIATION OF A FIELD LINE " as well. I would recommend watching it.
@PiyushSinghal-s6t
@PiyushSinghal-s6t 6 дней назад
I think its genuinely very obvious it is very fundamental just as( 1+1=2) A single question can't be rated good just becoz you somehow stuck there
@dnd2008yi
@dnd2008yi 10 дней назад
Yes, it will be proportional... I've taken a hypothetical hollow sphere around the charge q and intersected it through a plane passing through q... Now, if the plane also has some negligible thickness, then using solid angle concept, the flux through plane comes out to be Q/2Epsilon0 Which implies direct proportionality of flux with charge in that planar region
@Sonu-hv4ef
@Sonu-hv4ef 10 дней назад
Sir i think ye sawaal 150⁰ vertex angle ka cone leke solid angle se solve hua h indirectly jese vo question hota h 2 opposite charge wala line alpha angle pe nikli beeta angle pe enter hui... Further hum shayad agar ye prove kr de ki plane me kisi general point pe electric field strength proportional to charge hogi then BINGO ... btw big fan of your personality and problem solving skills, Sir❤
@vrishketusharma
@vrishketusharma 10 дней назад
will ask to saleem Bhaiya 🔥
@Esistomer
@Esistomer 10 дней назад
Sir, can I be your thumbnail designer? :)
@adityaroy-zg8xw
@adityaroy-zg8xw 10 дней назад
Padhle bhai jee nazdik aagya
@aryanchandra07
@aryanchandra07 10 дней назад
​@@adityaroy-zg8xwtereko kyse pata wo Jee ki tyyari kar rha hai 🤫
@adityaroy-zg8xw
@adityaroy-zg8xw 10 дней назад
@@aryanchandra07 tukka mara bhai
@Esistomer
@Esistomer 10 дней назад
@@adityaroy-zg8xw are bhai 11th jee me hu :)
@Aayan_200
@Aayan_200 10 дней назад
​@@adityaroy-zg8xw don't shout
@KeyserTheRedBeard
@KeyserTheRedBeard 3 дня назад
Great video, ISAC Learning (NMS Sir). Looking forward to your next upload! I smashed the thumbs up button on your content. Keep up the fantastic work. I'm curious, how do you think the deviation angle would change if we introduced a fourth charge into the system?
@NMSPhysicsISAC
@NMSPhysicsISAC 3 дня назад
@@KeyserTheRedBeard do subscribe for more such contents ❤
@k-editz69
@k-editz69 4 дня назад
mai uss offline ladke ke dost ka dost hu Mujhe bhi iss qn me doubt tha sir!
@KeshavSaini-g9i
@KeshavSaini-g9i 6 дней назад
sir jee main 2025 ke liye final test series kab launch hogi
@archnakumari9439
@archnakumari9439 9 дней назад
ru-vid.com2RaxpEXJkDc?si=XJynBQFL9e9QkthD In this a child used internal force yo change the state of motion or its just shift of centre of mass.
@mritunjaysingh956
@mritunjaysingh956 10 дней назад
Sir Agar ek point charge se N lines nikalti h , too vo N lines bhi to Har ek 2d plane me equally distribute hongi , too kya iss logic se nhi bool skte ki 2d plane me lines charge Ke proportional hongi .
@darshilmahajan5551
@darshilmahajan5551 10 дней назад
sir ijrs ka test dediya.... Batch milega ki nhi wo kese pata chalega??
@NMSPhysicsISAC
@NMSPhysicsISAC 10 дней назад
We'll contact you
@User213-z5f
@User213-z5f 9 дней назад
How can i also send you question related to physics sir?? Please reply..
@Lovenmssir
@Lovenmssir 9 дней назад
Telegram channel
@Roman-q3s7p
@Roman-q3s7p 10 дней назад
Sir ap ashsish Arora or insp nitin sachan sir ke bare mein kya opinion denge????
@Aayan_200
@Aayan_200 10 дней назад
Sir apne Bs dono cases me Field lines equate kri h? Though in case 1 ,u used EFL from single charge And in second case used Efl from all 3 charges Did u use only Angle proportionality and not Charge proportionality?
Далее
IIT-JEE Toppers: Where Are They Now?
16:07
Просмотров 1 млн
Я ИДЕАЛЬНО ПОЮ
00:31
Просмотров 476 тыс.
If you are NOT HAPPY, watch this | Abhi and Niyu
18:09
Просмотров 623 тыс.